SecondWind180
Thanks Received: 2
Jackie Chiles
Jackie Chiles
 
Posts: 29
Joined: October 03rd, 2013
 
 
 

Q9 - Governments have only one response

by SecondWind180 Mon Oct 21, 2013 4:16 am

Geeks dissect please. Narrowed it down to B and E. I chose E, but understand why B is correct.

Would E be correct had it said "If the cost of providing child care does not increase then child-care services are not regulated"

I'm pretty sure this (E) AC confuses Necc and Suff.

Thanks!
User avatar
 
tommywallach
Thanks Received: 468
Atticus Finch
Atticus Finch
 
Posts: 1041
Joined: August 11th, 2009
 
This post thanked 1 time.
 
 

Re: Q9 - Governments have only one response

by tommywallach Mon Oct 21, 2013 12:17 pm

Hey Secondwind,

Great question. Let's start by looking at the stimulus:

Public criticism of service --> regulation --> increased cost

Child care is publicly criticized --> there will be regulation

OKAY. So we can feel pretty confident that the correct answer is going to say something about cost.

(A) This is actually a pretty tricky answer, because the whole point of regulation is to improve the service. But we can't definitively say it'll happen.

(B) CORRECT. Yep. We know regulation leads to increased cost, and we know there will be regulation.

(C) This comes totally out of nowhere. Wuzzah?

(D) We don't have to infer this, because it's basically been stated already.

(E) This is what we call "reversed logic". Remember if A-->B, that doesn't mean -A-->-B (the correct contrapositive would be -B-->-A, the meaning of which would be "If the cost of a service hasn't gone up, it hasn't been subjected to regulation." -- Just like you said!).

Hope that helps!

-t
Tommy Wallach
Manhattan LSAT Instructor
twallach@manhattanprep.com
Image
 
sjy93528
Thanks Received: 0
Vinny Gambini
Vinny Gambini
 
Posts: 1
Joined: October 27th, 2014
 
 
 

Re: Q9 - Governments have only one response

by sjy93528 Mon Oct 27, 2014 7:26 am

Hey, But I am the one who choose D. Becase I think the inference questions are like the must be true, just choose the one matches the stimulus best. I see D is quite clear in the stimulus, so I choose it. So how to crack the inference questions basiclly?
User avatar
 
tommywallach
Thanks Received: 468
Atticus Finch
Atticus Finch
 
Posts: 1041
Joined: August 11th, 2009
 
 
 

Re: Q9 - Governments have only one response

by tommywallach Thu Oct 30, 2014 11:00 pm

Hey Sjy,

Yeah, that's my mistake. (D) is not in the statement. (D) is way too broad. We know that government is responding here because confidence has been totally undermined. But (D) does not include that part of the equation (i.e. voiced criticism --> confidence undermined --> government response), so it's wrong.

Hope that helps!

-t
Tommy Wallach
Manhattan LSAT Instructor
twallach@manhattanprep.com
Image
 
WilliamL952
Thanks Received: 0
Vinny Gambini
Vinny Gambini
 
Posts: 1
Joined: July 21st, 2017
 
 
 

Re: Q9 - Governments have only one response

by WilliamL952 Fri Jul 21, 2017 9:38 pm

I have a question about the stimulus. The first sentence says, "Governments have only one response to public criticism of socially necessary services, regulation of the activity providing these services."

Does this mean that whenever there exists public criticism of socially necessary services, the government MUST regulate the activity providing these services, or could they instead choose to do nothing, and hence not respond. Or would a non-response (the government not doing anything) still be considered a response?

This may seem irrelevant, but if answer choice D was phrased instead as, "If there exists public criticism of socially necessary services, the government is certain to respond," it would then force us to consider whether or not the government's choosing to say/do nothing is considered as a response.
 
JosephV
Thanks Received: 9
Jackie Chiles
Jackie Chiles
 
Posts: 38
Joined: July 26th, 2017
 
This post thanked 1 time.
 
 

Re: Q9 - Governments have only one response

by JosephV Mon Jan 15, 2018 3:42 pm

WilliamL952 Wrote:I have a question about the stimulus. The first sentence says, "Governments have only one response to public criticism of socially necessary services, regulation of the activity providing these services."

Does this mean that whenever there exists public criticism of socially necessary services, the government MUST regulate the activity providing these services, or could they instead choose to do nothing, and hence not respond. Or would a non-response (the government not doing anything) still be considered a response?


On the LSAT the question you raise is irrelevant. What I mean is that there is no place for guesswork. "They may act in this case but how do you know?" type of consideration does not exist. What the stimulus says is that if there is criticism the government will respond and the only thing they will do is regulate (not lower taxes, or build more highways, or anything else). As Tommy has diagrammed it above:

public criticism of socially necessary services ---> regulation ---> increased cost,

that is, you are certain that any time someone complains about an activity, the government will certainly (and all too happily, as reality shows) regulate the activity.

This may seem irrelevant, but if answer choice D was phrased instead as, "If there exists public criticism of socially necessary services, the government is certain to respond," it would then force us to consider whether or not the government's choosing to say/do nothing is considered as a response.


As I already said above, your version of answer choice (D) is exactly how you should approach the problem from the outset. Moreover, you must assume that any time there is criticism it will be followed by the government's regulating the activity in question.
 
JoP960
Thanks Received: 0
Vinny Gambini
Vinny Gambini
 
Posts: 7
Joined: July 01st, 2022
 
 
 

Re: Q9 - Governments have only one response

by JoP960 Mon Sep 12, 2022 1:27 pm

I think D is wrong is because there are two combined sufficient conditions (criticize very hard and the service is necessary )to trigger the necessary condition(the regulation is certain to response) in stimulus but D only said one of them. Is this right?